subject
Mathematics, 22.03.2021 20:40 baby0905rodriguez

at the gym, jasper was able to bench press 224 pounds, which was 7/8 of the weight that Balin was able to bench press. which shows the correct equation and value of x, the weight that Balin could bench press?​


at the gym, jasper was able to bench press 224 pounds, which was 7/8 of the weight that Balin was a

ansver
Answers: 2

Another question on Mathematics

question
Mathematics, 21.06.2019 14:30
The multiplication property of zero states that the product of any number and 0 is 0. what must be true about either a or b to make ab=0
Answers: 1
question
Mathematics, 21.06.2019 20:00
The function models the number of accidents
Answers: 1
question
Mathematics, 21.06.2019 23:00
Look at the chart showing the results of a baseball player at bat. which is the probability of getting a hit?
Answers: 1
question
Mathematics, 22.06.2019 00:30
For the word below, click on the drop-down arrows to select the root and its meaning. version
Answers: 2
You know the right answer?
at the gym, jasper was able to bench press 224 pounds, which was 7/8 of the weight that Balin was ab...
Questions
Questions on the website: 13722363